Đến nội dung

Hình ảnh

Prove: Tồn tại m thỏa $\left( {{m^3} + 17} \right) \vdots {3^n}$


  • Please log in to reply
Chủ đề này có 3 trả lời

#1
NLT

NLT

    Trung úy

  • Hiệp sỹ
  • 871 Bài viết
$\boxed{\text{NLT_CL}}$ Exercise: Chứng minh rằng với mọi số tự nhiên n không nhỏ hơn 2, tồn tại một số tự nhiên m sao cho $\left( {{m^3} + 17} \right) \vdots {3^n}$, nhưng $\left( {{m^3} + 17} \right)$ không chia hết cho ${3^{n + 1}}$. :icon12:
___

Bài viết đã được chỉnh sửa nội dung bởi Nguyen Lam Thinh: 17-06-2012 - 08:20

GEOMETRY IS WONDERFUL !!!

Some people who are good at calculus think that they will become leading mathematicians. It's funny and stupid.


Nguyễn Lâm Thịnh

#2
nguyenta98

nguyenta98

    Thượng úy

  • Hiệp sỹ
  • 1259 Bài viết

$\boxed{\text{NLT_CL}}$ Exercise: Chứng minh rằng với mọi số tự nhiên n không nhỏ hơn 2, tồn tại một số tự nhiên m sao cho $\left( {{m^3} + 17} \right) \vdots {3^n}$, nhưng $\left( {{m^3} + 17} \right)$ không chia hết cho ${3^{n + 1}}$. :icon12:
___

Giải như sau:
Ta sẽ chứng minh bằng quy nạp toán học
Thử với $n=1$ thì $m=4$ khi đó $m^3+17 \vdots 3$ mà không chia hết cho $9$ thỏa mãn
Thử với $n=2$ tương tự cũng thỏa mãn
Giả sử $n=k$ đúng hay tồn tại $m$ để $m^3+17 \vdots 3^k$ mà $m^3+17 \not \vdots 3^{k+1}$
Ta sẽ chứng minh $n=k+1$ cũng đúng hay tồn tại số $t$ để $t^3+17 \vdots 3^{k+1}$ mà $\not \vdots 3^{k+2}$
Thật vậy, từ GTQN suy ra $m^3+17=3^n.q$ mà do $m^3+17 \not \vdots 3^{n+1}$ suy ra $gcd(q,3)=1$
Do đó $q$ chỉ có dạng $3k+1$ hoặc $3k+2$
TH1: $q=3k+1$ khi ấy $m^3+17=3^n.(3k+1)$
  • Nếu $k=3u \Rightarrow m^3+17=3^n.(9u+1)$
Do đó chọn $t=v.3^{n-1}+m$ suy ra khi ấy
$t^3+17=(v.3^{n-1}+m)^3+17=v^3.3^{3(n-1)}+3.m^2.(v.3^{n-1})+3.m.(v.3^{n-1})^2+m^3+17$
$=v^3.3^{3(n-1)}+3.m^2.(v.3^{n-1})+ 3.m.v^2.3^{2n-2}+3^n(9u+1)$
$=v^3.3^{3(n-1)}+m^2.v.3^{n}+ m.v^2.3^{2n-1}+3^n(9u+1)$
Nhận thấy do $n>2$ nên $3(n-1)>n+1 \Rightarrow v^3.3^{3(n-1)} \vdots 3^{n+1}$ và dễ dàng suy ra $3(n-1)>n+2$ nên $v^3.3^{3(n-1)}$ cũng chia hết cho $3^{n+2}$ $(1)$
Và $n>2$ nên $2n-1>n+1 \Rightarrow m.v^2.3^{2n-1} \vdots 3^{n+1}$ và dễ dàng suy ra $2n-1>n+2$ nên $v^3.3^{3(n-1)}$ cũng chia hết cho $3^{n+2}$ $(2)$
Giờ ta xét $m^2.v.3^n+3^n(9u+1)$
Nhận thấy do $m^3+17=3^n.(9u+1)$ mà do $n>2$ cho nên $m^3+17 \vdots 9 \rightarrow m^3 \equiv 1 \pmod{9} \rightarrow m \equiv 1,4,7 \pmod{9}$
Suy ra $m^2 \equiv 1,4,7 \pmod{9}$
Quay trở lại $m^2.v.3^n+3^n(9u+1)=3^n(v.m^2+9u+1)$
Nếu $m^2 \equiv 1 \pmod{9}$ thì khi đó $m^2=9p+1$ lúc đó chọn $v=2$ khi ấy $3^n(v.m^2+9u+1)=3^n(2.(9p+1)+9u+1)=3^n.(18p+9u+3)=3^{n+1}.(6p+3u+1)$
Do đó $3^{n+1}.(6p+3u+1) \vdots 3^{n+1}$ mà không chia hết cho $3^{n+2}$ (do $gcd(6p+3u+1,3)=1$) $(3)$
Từ $(1)(2)(3)$ suy ra $t=2.3^{n-1}+m$ là số thỏa mãn $t^3+17$ thỏa đề
Nếu $m^2 \equiv 4 \pmod{9}$ thì khi đó $m^2=9p+4$ lúc đó chọn $v=5$ lúc đó $3^n.(v.m^2+9u+1)=3^n(45p+9u+21)=3^{n+1}(15p+3u+7)$
Và chứng minh tương tự $3^{n+1}(15p+3u+7) \vdots 3^{n+1}$ mà không chia hết cho $3^{n+2}$ (do $gcd(15p+3u+1,3)=1$) $(4)$
Từ $(1)(2)(4)$ suy ra $t=5.3^{n-1}+m$ là số thỏa mãn $t^3+17$ thỏa đề
Nếu $m^2 \equiv 7 \pmod{9}$ khi đó $m^2=9p+7$ lúc đó chọn $v=2$ lúc đó $3^n.(v.m^2+9u+1)=3^n(18p+9u+15)=3^{n+1}(6p+3u+5)$
Chứng minh tương tự $3^{n+1}(6p+3u+5) \vdots 3^{n+1}$ mà không chia hết cho $3^{n+2}$ (do $gcd(6p+3u+5,3)=1$) $(5)$
Từ $(1)(2)(5)$ suy ra $t=2.3^{n-1}+m$ là số thỏa mãn $t^3+17$ thỏa đề
Như vậy trường hợp $m^3+17=3^n.(9u+1)$ được chứng minh
  • Nếu $k=3u+1$ khi đó $m^3+17=3^n.(9u+4)$ chứng minh hoàn toàn tương tự và ta chọn $v=2,2,5$ tương ứng với $m^2 \equiv 1,4,7 \pmod{9}$ có đpcm
  • Nếu $k=3u+3$ khi đó $m^3+17=3^n.(9u+7)$ chứng minh hoàn toàn tương tự và ta chọn $v=5,2,2$ tương ứng với $m^2 \equiv 1,4,7 \pmod{9}$ có đpcm
TH2: $q=3k+2$ khi đó $m^3+17=3^n.(3k+2)$
Cũng xét tương tự 3 trường hợp con $k=3u,3u+1,3u+2$ với phương pháp chọn $v$ tương tự ta cũng có đpcm
Tóm lại bài toán được giải hoàn toàn, cách chứng minh quy nạp đã thành công trong bài toán này, vì sao chúng ta phải chọn $v$ thích hợp? Mục đích để $3^n.(v.m^2+9u+1)$ và $3^n.(v.m^2+9u+4)$ và $3^n.(v.m^2+9u+7)$ chia hết cho $3^{n+1}$ mà không chia hết cho $3^{n+2}$ và như vậy sẽ làm bài toán được giải quyết

P/S mất cả ngày nghĩ bài này :( chú ý rằng mấy phần chứng minh tương tự, phương pháp làm y hệt và chắc chắn sẽ đến kết quả, nếu cần mình sẽ post nhưng chỉ sợ dài thôi :D

Bài viết đã được chỉnh sửa nội dung bởi nguyenta98: 17-06-2012 - 22:39


#3
NLT

NLT

    Trung úy

  • Hiệp sỹ
  • 871 Bài viết

Giải như sau:
P/S mất cả ngày nghĩ bài này :( chú ý rằng mấy phần chứng minh tương tự, phương pháp làm y hệt và chắc chắn sẽ đến kết quả, nếu cần mình sẽ post nhưng chỉ sợ dài thôi :D

P/S: Nguyên xem lại, bài này còn một số lỗi nho nhỏ, nhưng cũng không đáng kể lắm :)
________________

$\boxed{\text{NLT_CL}}$ Another Solution:

Với $n=2$, dễ thấy tồn tại $m$ thỏa mãn.

Ta cũng chứng minh bài này bằng quy nạp.

Giả sử $n=k$ đúng hay tồn tại $m$ để $m^3+17 \vdots 3^k$ mà $m^3+17$ không chia hết cho $3^{k+1}$
Ta chứng minh rằng đối với trường hợp $n=k+1$ cũng đúng tức là tồn tại một số $m'$ sao cho $m'^3+17 \vdots 3^{k+1}$ mà không chia hết cho $3^{k+2}$.

Đặt ${m^3} + 17 = {3^k}.n$ $\to n $ không chia hết cho 3.

$ \Rightarrow \left[ \begin{array}{l}
n \equiv 2 \\
n \equiv 1 \\
\end{array} \right.\left( {\bmod 3} \right) \Rightarrow \left[ \begin{array}{l}
{m^3} + 17 \equiv {2.3^k} \\
{m^3} + 17 \equiv {3^k} \\
\end{array} \right.\left( {\bmod {3^{k + 1}}} \right)$.

$\bullet$ Trường hợp 1: ${m^3} + 17 \equiv 2.{3^k} (mod 3^{k+1})$

Xét: ${\left( {m + {3^{k - 1}}} \right)^3} = {m^3} + {m^2}{3^k} + m{3^{2k - 1}} + {3^{3k - 3}} \equiv {m^3} + {m^2}{3^k}\left( {\bmod {3^{k + 1}}} \right)$.
(Do $k \ge 2 \to {3^{2k - 1}} \vdots {3^{k + 1}};{3^{3k - 3}} \vdots {3^{k + 1}}$).

Suy ra: ${\left( {m + {3^{k - 1}}} \right)^3} + 17 \equiv {m^3} + {m^2}{3^k} + 17 \equiv {2.3^k} + {m^2}{3^k} \equiv 0\left( {\bmod {3^{k + 1}}} \right)$ (vì $m$ không chia hết cho 3 nên ${m^2} \equiv 1\left( {\bmod 3} \right) \to 2 + {m^2} \vdots 3 \to \left( {2 + {m^2}} \right){.3^k} \vdots {3^{k + 1}}$).

Như vậy, ở trường hợp 1, ta có: ${\left( {m + {3^{k - 1}}} \right)^3} + 17 \vdots {3^{k + 1}}$.

$\bullet$ Trường hợp 2: ${m^3} + 17 \equiv {3^k} (mod 3^{k+1})$.

Xét: ${\left( {m - {3^{k - 1}}} \right)^3} = {m^3} - {m^2}{3^k} + m{3^{2k - 1}} - {3^{3k - 3}} \equiv {m^3} - {m^2}{3^k}\left( {\bmod {3^{k + 1}}} \right)$.
(Do $k \ge 2 \to {3^{2k - 1}} \vdots {3^{k + 1}};{3^{3k - 3}} \vdots {3^{k + 1}}$).

Suy ra: ${\left( {m - {3^{k - 1}}} \right)^3} + 17 \equiv {m^3} - {m^2}{3^k} + 17 \equiv {3^k} - {m^2}{3^k} \equiv 0\left( {\bmod {3^{k + 1}}} \right)$
(vì $m$ không chia hết cho 3 nên ${m^2} \equiv 1\left( {\bmod 3} \right) \to 1 - {m^2} \vdots 3 \to \left( {1 - {m^2}} \right){.3^k} \vdots {3^{k + 1}}$.

Như vậy, ở trường hợp 2 ta có: ${\left( {m - {3^{k - 1}}} \right)^3} + 17 \vdots {3^{k + 1}}$.


Để ý rằng, $m$ không chia hết cho 3 (do $m^3 + 17$ chia hết cho 3) nên ${\left( {m + {3^{k - 1}}} \right)^3}\& {\left( {m - {3^{k - 1}}} \right)^3}$đều không chia hết cho 3.

Tóm lại, ta đều tìm được số nguyên t không chia hết cho 3 mà ${t^3} + 17 \vdots {3^{k + 1}}$.

* Nếu ${t^3} + 17$ không chia hết cho $3^{k + 2}$ thì vấn đề đã được giải quyết.

* Nếu ${t^3} + 17 \vdots {3^{k + 2}}$, ta xét số $p = t - {2.3^k}$.

Ta có: ${\left( {t - {{2.3}^k}} \right)^3} = {t^3} - 2{t^2}{.3^{k+1}} + {2.3^{2k + 1}} - {8.3^{3k}} \equiv {t^3} - 2{t^2}{.3^{k+1}}\left( {\bmod 3^{k + 2}} \right)$ (do $3k \ge k + 2;2k + 1 \ge k + 2$).
$ \to {p^3} \equiv {t^3} - 2{t^2}{.3^{k + 1}}\left( {\bmod 3^{k + 2}} \right) \to {p^3} + 17 \equiv {t^3} - 2{t^2}{.3^{k + 1}} + 17 \equiv - 2{t^2}{.3^{k + 1}}\left( {\bmod 3^{k+2} } \right)$.
Mặt khác $t$ không chia hết cho 3 nên $ - 2{t^2}{.3^{k + 1}}$ không chia hết cho $3^{k + 2}$, do vậy $p^3+17$ không chia hết cho $3^{k+2}$.

Dễ thấy $p$ không chia hết cho 3. Để ý rằng: ${p^3} = {\left( {t - {{2.3}^k}} \right)^3} = {t^3} - 2{t^2}{.3^{k + 1}} + {2.3^{2k + 1}} - {8.3^{3k}} \equiv {t^3}\left( {\bmod {3^{k + 1}}} \right) \to {p^3} + 17 \equiv {t^3} + 17\left( {\bmod {3^{k + 1}}} \right)
\to {p^3} + 17 \vdots {3^{k + 1}}$.

Như vậy, ở trường hợp ${t^3} + 17 \vdots {3^{k + 2}}$ ta tìm được một số $p$ thỏa ${p^3} + 17 \vdots {3^{k + 1}}$ mà không chia hết cho $3^{k + 2}$.

Tóm lại, ta đã chứng minh được vấn đề đúng trong trường hợp $n=k+1$.

Theo nguyên lý quy nạp $\to Q.E.D $. :icon12:

$\boxed{\textit{The problem is completely solved ...}}$.
___
P/S: Bài làm không tránh khỏi những sai sót và lập luận còn chưa chặt chẽ, các bạn thông cảm và góp ý nhé, thanks ! :)
___

Bài viết đã được chỉnh sửa nội dung bởi yeutoan11: 18-06-2012 - 18:22

GEOMETRY IS WONDERFUL !!!

Some people who are good at calculus think that they will become leading mathematicians. It's funny and stupid.


Nguyễn Lâm Thịnh

#4
nguyenta98

nguyenta98

    Thượng úy

  • Hiệp sỹ
  • 1259 Bài viết

$\boxed{\text{NLT_CL}}$ Exercise: Chứng minh rằng với mọi số tự nhiên n không nhỏ hơn 2, tồn tại một số tự nhiên m sao cho $\left( {{m^3} + 17} \right) \vdots {3^n}$, nhưng $\left( {{m^3} + 17} \right)$ không chia hết cho ${3^{n + 1}}$. :icon12:
___

Mấu chốt bài này là ở bổ đề:
Bổ đê:
Cho $gcd(a,q)=1, gcd(b,q)=1$ khi ấy luôn luôn tồn tại $k$ sao cho
$ak+b \vdots q$ hoặc $ak-b \vdots q$ hoặc $b-ak \vdots q$

Cm: Cực kì đơn giản, ta cm đại diện tồn tại $k$ sao cho $b-ak \vdots q$
Gọi $b=qe+f, 1\le f\le q-1$ và $a=qm+n, 1\le n\le q$ và do $gcd(b,q)=gcd(a,q)=1 \Rightarrow gcd(b,f)=gcd(b,n)=1$ $(1)$
Khi ấy $b-ak \equiv f-an \pmod{q}$
Xét dãy $A=f-n,f-2n,f-3n,...f-qn$
Nhận thấy trong dãy $A$ không tồn tại hai số nào có cùng số dư khi chia $q$
Vì giả sử trái lại suy ra $f-hn \equiv f-gn \pmod{q} (\text{với h<g} \Rightarrow q|f-hn-(f-gn) \Rightarrow q|(g-h)n$
Và do $(1)$ nên suy ra $gcd(q,n)=1$ do đó $q|g-h$ nhưng đây là điều vô lý do $1\le g,h\le q$
Do đó trong dãy $A$ không có hai số nào cùng số dư hay nói cách khác nó là một Hệ Thặng Dư đầy đủ $mod(q)$
Do đó ắt tồn tại một phân tử chia hết cho $q$
Hay tồn tại $f-ka \vdots q$ do đó tồn tại $k$ thỏa mãn $b-ak \vdots q$ suy ra $đpcm$

Và cách của minh trên chính là bổ đề này, số $v$ là số sẽ phải tìm để thỏa mãn :D
Lời giải tổng quát tại
http://diendantoanho...=0




0 người đang xem chủ đề

0 thành viên, 0 khách, 0 thành viên ẩn danh